Đến nội dung

Hình ảnh

Topic ôn thi hình học vào cấp 3 chuyên

hình học

  • Please log in to reply
Chủ đề này có 239 trả lời

#221
Drago

Drago

    Sĩ quan

  • Thành viên
  • 462 Bài viết

Lời giải bài toán 78.

Bổ đề  Cho tam giác $ABC$ có tia phân giác trong $AD$. Trên đoạn thẳng $AD$ lấy các điểm $K,L$ ($K$ nằm giữa $A$ và $L$) thỏa mãn $\angle{ACK}=\angle{LCB}$. Chứng minh rằng $\angle{ABK}=\angle{LBC}$

attachicon.gifScreenHunter_21 May. 04 16.29.jpg

(Bổ đề khá quen thuộc nên mình không chứng minh). 

Spoiler

 

Quay trở lại bài toán

attachicon.gifScreenHunter_19 May. 04 16.18.jpg

Vẽ đường kính $PW$ của $(O)$. Gọi trung điểm của $BC$ là $N$. Bài toán cần chứng minh trở thành chứng minh tứ giác $BKNM$ nội tiếp. Dễ chứng minh được $N$ thuộc đường tròn đường kính $PC$.

Ta có: $\angle{ACL}=\angle{CLW}-\angle{LAC}=\angle{CLW}-\frac{\hat{A}}{2}$  $(1)$

Mặt khác $\angle{KCN}=\angle{KPN}=\angle{KPC}-\angle{WBC}$

Mà $\angle{WBC}=\frac{\hat{A}}{2}$ và $\angle{KPC}=\angle{CLW}$

$\Rightarrow \angle{KCN}=\angle{CLW}-\frac{\hat{A}}{2}$ $(2)$

Từ $(1)$ và $(2)$ suy ra $\angle{KCN}=\angle{ACL}$ $\Rightarrow \angle{ACK}=\angle{LCB}$

Áp dụng bổ đề thì ta có ngay $\angle{ABK}=\angle{LBC}$  $(3)$

                                              $\Rightarrow \angle{ABK}=\angle{NBM}$

                                              $\Rightarrow \angle{ABN}=\angle{KBM}$

Mà $\angle{ABN}=\angle{AWC}$ (cùng chắn $AC$)

Nên $\angle{AWC}=\angle{KBM}$  $(4)$

Từ $(3)$ ta cũng có $\angle{BKW}=\angle{ABK}+\frac{\hat{A}}{2}=\angle{LBC}+\angle{CBW}=\angle{LBW}$

$\Rightarrow \angle{BKW}=\angle{LBW}$

$\Rightarrow \triangle{BLW} \sim \triangle{KBW}$

$\Rightarrow \frac{BW}{BL}=\frac{KW}{BK}$

$\Rightarrow \frac{WC}{BM}=\frac{KW}{BK}$ $(BL=BM,BW=WC)$  

$\Rightarrow \frac{WC}{KW}=\frac{BM}{BK}$ $(5)$

Từ $(4)(5)$ suy ra $\triangle{BKM} \sim \triangle{WKC}$

                 $\Rightarrow \angle{BKM}=\angle{WKC}$

Mặt khác $\angle{WKC}=\angle{LNB}=\angle{BNM}$ (Vì tứ giác $BLNC$ nội tiếp và $M,L$ đối xứng qua $BC$)

Suy ra $ \angle{BKM}=\angle{BNM}$ hay tứ giác $BKNM$ nội tiếp tức là $(BKM)$ đi qua trung điểm $N$ của đoạn thẳng $BC$ (đpcm)

Spoiler

 

Lời giải bài toán 78.

Bổ đề  Cho tam giác $ABC$ có tia phân giác trong $AD$. Trên đoạn thẳng $AD$ lấy các điểm $K,L$ ($K$ nằm giữa $A$ và $L$) thỏa mãn $\angle{ACK}=\angle{LCB}$. Chứng minh rằng $\angle{ABK}=\angle{LBC}$

attachicon.gifScreenHunter_21 May. 04 16.29.jpg

(Bổ đề khá quen thuộc nên mình không chứng minh). 

Spoiler

 

Quay trở lại bài toán

attachicon.gifScreenHunter_19 May. 04 16.18.jpg

Vẽ đường kính $PW$ của $(O)$. Gọi trung điểm của $BC$ là $N$. Bài toán cần chứng minh trở thành chứng minh tứ giác $BKNM$ nội tiếp. Dễ chứng minh được $N$ thuộc đường tròn đường kính $PC$.

Ta có: $\angle{ACL}=\angle{CLW}-\angle{LAC}=\angle{CLW}-\frac{\hat{A}}{2}$  $(1)$

Mặt khác $\angle{KCN}=\angle{KPN}=\angle{KPC}-\angle{WBC}$

Mà $\angle{WBC}=\frac{\hat{A}}{2}$ và $\angle{KPC}=\angle{CLW}$

$\Rightarrow \angle{KCN}=\angle{CLW}-\frac{\hat{A}}{2}$ $(2)$

Từ $(1)$ và $(2)$ suy ra $\angle{KCN}=\angle{ACL}$ $\Rightarrow \angle{ACK}=\angle{LCB}$

Áp dụng bổ đề thì ta có ngay $\angle{ABK}=\angle{LBC}$  $(3)$

                                              $\Rightarrow \angle{ABK}=\angle{NBM}$

                                              $\Rightarrow \angle{ABN}=\angle{KBM}$

Mà $\angle{ABN}=\angle{AWC}$ (cùng chắn $AC$)

Nên $\angle{AWC}=\angle{KBM}$  $(4)$

Từ $(3)$ ta cũng có $\angle{BKW}=\angle{ABK}+\frac{\hat{A}}{2}=\angle{LBC}+\angle{CBW}=\angle{LBW}$

$\Rightarrow \angle{BKW}=\angle{LBW}$

$\Rightarrow \triangle{BLW} \sim \triangle{KBW}$

$\Rightarrow \frac{BW}{BL}=\frac{KW}{BK}$

$\Rightarrow \frac{WC}{BM}=\frac{KW}{BK}$ $(BL=BM,BW=WC)$  

$\Rightarrow \frac{WC}{KW}=\frac{BM}{BK}$ $(5)$

Từ $(4)(5)$ suy ra $\triangle{BKM} \sim \triangle{WKC}$

                 $\Rightarrow \angle{BKM}=\angle{WKC}$

Mặt khác $\angle{WKC}=\angle{LNB}=\angle{BNM}$ (Vì tứ giác $BLNC$ nội tiếp và $M,L$ đối xứng qua $BC$)

Suy ra $ \angle{BKM}=\angle{BNM}$ hay tứ giác $BKNM$ nội tiếp tức là $(BKM)$ đi qua trung điểm $N$ của đoạn thẳng $BC$ (đpcm)

Spoiler

Chứng minh 2 tam giác đồng dạng trong bổ đề ntn bạn NHoang.


$\mathbb{VTL}$


#222
Drago

Drago

    Sĩ quan

  • Thành viên
  • 462 Bài viết

Mình thấy đâu cần AE là phân giác $\angle{BAD}$ đâu nhỉ, E bất kì chuyển động trên BC vẫn làm được mà.  :wacko:

attachicon.gifgeogebra-export (1).png

Lời giải bài 60:

Dễ dàng chứng minh được BDFC là hình thang cân. 

mà ABCD là hình bình hành nên $\angle{BFD}=\angle{BCD}=\angle{BAD}$ và $\angle{FDB}=\angle{DBC}=\angle{ADB}$

do đó $\Delta ADB=\Delta FDB \Rightarrow$ A và F đối xứng với nhau qua BD

Từ đó ta có $AF\perp BD$

Lại có $BD\parallel FC$ nên $AF\perp FC$ hay $\angle{AFC}=90^0$

Phiền bạn giải thích giúp mình vì sao BDFC là hình thang cân. :))


$\mathbb{VTL}$


#223
Nguyenphuctang

Nguyenphuctang

    Sĩ quan

  • Banned
  • 499 Bài viết

Chứng minh 2 tam giác đồng dạng trong bổ đề ntn bạn NHoang.

Link: 
https://artofproblem...h570912p3372652

Post lại cái hình cho dễ nhìn : 

18194600_312495132513884_294548017903064


Bài viết đã được chỉnh sửa nội dung bởi Nguyenphuctang: 02-06-2017 - 10:43


#224
Drago

Drago

    Sĩ quan

  • Thành viên
  • 462 Bài viết

Link: 
https://artofproblem...h570912p3372652

Post lại cái hình cho dễ nhìn : 

18194600_312495132513884_294548017903064

Trong bài viết mà bạn dẫn chỉ dẫn lại và bổ đề không chứng minh và lời giải bài đó thôi. Minh muốn hỏi về cách chứng minh bổ đề ấy. Cảm ơn bạn


$\mathbb{VTL}$


#225
truongquangtuann

truongquangtuann

    Lính mới

  • Thành viên mới
  • 6 Bài viết

Cho đường tròn (O) đường kính AB . Lấy I là điểm bất kì trên đoạn thẳng AO . Kẻ dây cung CD của (O) qua I và vuông góc với AB. Lấy E là điểm trên (O) sao cho D là điểm chính giữa của cung nhỏ AE. Gọi K là giao điểm của AE và CD . b) Đường thẳng qua I song song với CE , cắt AE,BE lần lượt tại P,Q. Chứng minh DPEQ là hình chữ nhật

c) GỌi K là giao điểm của AE và CD . TÌm vị trí của điểm I trên AB để tích IK.ID đạt max

( Đề Chuyên Tin Amsterdam 2015) 



#226
truongquangtuann

truongquangtuann

    Lính mới

  • Thành viên mới
  • 6 Bài viết

Cho tam giác ABC có ba góc nhọn nội tiếp trong đường tròn (O) các đường cao AM,BN ,CP của tam giác ABC cắt nhau tại H.

2) Trên cung nhỏ BC lấy điểm Q . GỌi E và F thứ tự là các điểm đối xứng của Q qua đường thẳng AB và AC . Chứng minh : 3 điểm E,H,F thẳng hàng 

3)Gọi J là giao điểm của QE và AB , I là giao điểm của QF và AC . Tìm vị trí của diểm Q trên cung nhỏ BC để AB/QJ+AC/AI nhỏ nhất

( Đề chuyên toán Amsterdam 2015)



#227
Naix

Naix

    Lính mới

  • Thành viên
  • 8 Bài viết

Trong bài viết mà bạn dẫn chỉ dẫn lại và bổ đề không chứng minh và lời giải bài đó thôi. Minh muốn hỏi về cách chứng minh bổ đề ấy. Cảm ơn bạn

Thực ra bài này dùng bổ đề trên là không cần thiết và phức tạp, có thể biến đổi góc gọn hơn như sau (bạn tự nhìn hình ở trên)

\[\angle LBQ+\angle LQB=\angle KBA+\angle LKC=\angle KBA+\angle LKQ+\angle QKC\] Với chú ý rằng $\angle QKC=\angle QPC=\angle BAK,$ từ đó \[ \angle LBQ+\angle LQB=\angle KBA+\angle KAB+\angle LKQ=\angle BKQ\] Do đó $\angle BKQ+\angle BMQ=180^{\circ}$ suy ra đpcm.



#228
Nguyen Xuan Hieu

Nguyen Xuan Hieu

    Binh nhất

  • Thành viên mới
  • 39 Bài viết

Bài 108:Cho 2 đường tròn (O,R) và (O',R') cắt nhau tại A và B. Trên tia đối của AB lấy M, từ M vẽ các tiếp tuyến MC,MD với (O') và D thuộc miền trong của (O). AC, AD cắt (O) tại P,Q chứng minh P và Q luôn đi qua một điểm cố định.



#229
trinhhoangdung123456

trinhhoangdung123456

    Hạ sĩ

  • Thành viên
  • 88 Bài viết

 Bài 109 : Cho đường tròn (O; R) và đường thẳng d không đi qua O cắt đường thẳng (O) tại hai điểm A và B. Từ 1 điểm M tùy ý trên đường thẳng d và ở ngoài đường tròn (O) vẽ 2 tiếp tuyến MN và MP với đường tròn (O) ( M, N là 2 tiếp điểm ).

a) CMR $MN^{2}=MP^{2}=MA.MB$

b) Dựng vị trí điểm M trên đường thẳng d sao cho tứ giác MNOP là hình vuông.

c) CMR tâm của đường tròn nội tiếp và tâm của đường tròn ngoại tiếp tam giác MNP lần lượt chạy trên 2 đường cố định khi M di động trên đường thẳng d.



#230
azAZ

azAZ

    Lính mới

  • Thành viên mới
  • 7 Bài viết

1.Cho tam giác ABC có 3 góc nhọn. Các đường cao AD, BE, CF cắt nhau tại H. Gọi K là điểm tùy ý trên cạnh BC.

L là hình chiếu của H trên AK. Chứng minh các tứ giác BFLK và CELK nội tiếp

 

2.Cho tam giác ABC có 3 góc nhọn. Các đường cao AD, BE, CF cắt nhau tại H. Gọi K là điểm tùy ý trên cạnh BC (K khác B, C, D).

Đường tròn ngoại tiếp tam giác CEK và tam giác BFK cắt nhau tại L.

a) Chứng minh A, L,  K thẳng hàng

 

b) Chứng minh HL vuông góc với AK

 

3. Cho tam giác ABC có 3 góc nhọn. Các đường cao AD, BE, CF cắt nhau tại H. Gọi K là điểm tùy ý trên cạnh BC (K khác B, C).

Kẻ đường kính KM của đường tròn ngoại tiếp tam giác BKF và đường kính KN của đường tròn ngoại tiếp tam giác CEK.

Chứng minh M, H, K thẳng hàng

 

4. Cho tam giác ABC có 3 góc nhọn. Các đường cao AD, BE, CF cắt nhau tại H. Gọi K là điểm tùy ý trên cạnh BC (K khác B, C).

Đường tròn ngoại tiếp tam giác BKF và đường tròn ngoại tiếp tam giác CEK cắt nhau tại N.

Tìm vị trí của K trên BC để BC, EF, HL đồng quy.

 
Nguồn: 
Câu hỏi của Nguyễn Huệ Lam - Toán lớp 9 - Học toán với OnlineMath

Bài viết đã được chỉnh sửa nội dung bởi azAZ: 17-12-2017 - 14:16


#231
Olympusreacher

Olympusreacher

    Hạ sĩ

  • Thành viên
  • 53 Bài viết

Mình xin đóng góp 1 bài (Sputnik hình học):

Cho tia $Ax$ và một điểm $E$ khác điểm $A$, $E \epsilon Ax$. Từ $E$ vẽ tia $Ay$. Hai điểm $C,D$ phân biệt, khác điểm $E$, cho trước trên tia $Ey$. Một điểm $B$ chạy trên tia $Ax$. Các đường thẳng $AC$ và $BD$ cắt nhau ở $M$, $AD$ và $BC$ cắt nhau ở $N$.

a) Chứng minh rằng đường thẳng $MN$ luôn cắt tia $Ey$ tại một điểm $F$ cố định.

b) Hãy xác định một vị trí của điểm $B$ trên tia $Ex$ sao cho các tam giác $MCD$ và $NCD$ tương ứng có diện tích bằng nhau.


Weak people revenge, strong people forgive, intelligent people ignore.

                                                                                                                                                                   Albert Einstein


#232
xuanhoan23112002

xuanhoan23112002

    Trung sĩ

  • Thành viên
  • 103 Bài viết

Bài 19 (Đề thi vào lớp 10 chuyên Nam Định 2017):Cho tam giác ABC có ba góc nhọn nội tiếp đường tròn (O), AB < AC. Các tiếp tuyến của đường tròn (O) tại B và C cắt nhau tại M. Đường thẳng qua M song song với AB cắt đường tròn (O) tại D và E (D thuộc cung nhỏ BC), cắt BC tại F, cắt AC tại I.

1) Chứng minh năm điểm M, B, O, I, C cùng thuộc một đường tròn

2)Chứng minh$\frac{FI}{FE}=\frac{FD}{FM}$

3) OI cắt (O) tại P và Q (P thuộc cung nhỏ AB). QF cắt (O) tại T (T khác Q). Tính tỉ số $\frac{TQ^2+TM^2}{MQ^2}$



#233
Minhcamgia

Minhcamgia

    Thượng sĩ

  • Thành viên
  • 227 Bài viết

Bài 19 (Đề thi vào lớp 10 chuyên Nam Định 2017):Cho tam giác ABC có ba góc nhọn nội tiếp đường tròn (O), AB < AC. Các tiếp tuyến của đường tròn (O) tại B và C cắt nhau tại M. Đường thẳng qua M song song với AB cắt đường tròn (O) tại D và E (D thuộc cung nhỏ BC), cắt BC tại F, cắt AC tại I.

1) Chứng minh năm điểm M, B, O, I, C cùng thuộc một đường tròn

2)Chứng minh$\frac{FI}{FE}=\frac{FD}{FM}$

3) OI cắt (O) tại P và Q (P thuộc cung nhỏ AB). QF cắt (O) tại T (T khác Q). Tính tỉ số $\frac{TQ^2+TM^2}{MQ^2}$

Lời giải bài 19.

1) Ta có $\angle OBM  = \angle OCM = 90 \Rightarrow OBCM$ nội tiếp đường tròn đường kính $OM$ 

Do $MI \parallel AB \Rightarrow \angle MIC =  \angle BAC = \frac{1}{2} \angle BOC = \angle MOB = \angle MOC \Rightarrow MOIC$ nội tiếp đường tròn đường kính $OM$ suy ra $bB,C,O,I,M$ nội tiếp đường tròn đường kính $OM$.

2) Theo hệ thức lượng trong đường tròn ta có $FE.FD = FB.FC = FI.FM \Rightarrow FI.FM = FD.FE \Rightarrow \frac{FI}{FE} = \frac{FD}{FM}$.

3) Ta có $\angle OIM = \angle OCM = 90$ mà theo hệ thức lượng trong đường tròn ta lại có $FQ.FT = FB.FC = FI.FM \Rightarrow FI.FM = FT.FQ \Rightarrow TIQM$ nội tiếp $\Rightarrow \angle MTQ = \angle MIQ = \angle MIO = 90 \Rightarrow \Delta MTQ$ vuông tại $M$ suy ra $MQ^2 = MT^2 + TQ^2 \Rightarrow \frac{MT^2 + TQ^2}{MQ^2} = 1$.

Hình gửi kèm

  • diendan(7).PNG


#234
yeutoan89

yeutoan89

    Binh nhì

  • Thành viên mới
  • 15 Bài viết

Bài 110: cho tam giác ABC nhọn(AB<AC) nội tiếp đường tròn tâm (O). Gọi I là tâm đường tròn nội tiếp tam giác ABC. Tia AI cắt (O) tại J khác A. Đường thẳng JO cắt (O) tại K khác J và cắt BC tại E.

a.Chứng minh rằng J là tâm đường tròn ngoại tiếp tam giác IBC và JE.JK=JI2

b.Tiếp tuyến của (O) tại B và C cắt nhau tại S. Chứng minh rằng SJ.EK=SK.EJ
c.Đường thẳng SA cắt (O) tại D khác A ,đường thẳng DI cắt (O) tại M khác D
.Chứng minh rằng JM đi qua trung điểm của đoạn thẳng IE .
Bài 111: Cho tứ giác lồi ABCD . Gọi M,N lần lượt là trung điểm của AD và
BC ;AN cắt BM tại P , DN cắt CM tại Q .Tìm giá trị nhỏ nhất của biểu thức
$\ \frac{NA}{NP}+\frac{ND}{NQ}+\frac{MB}{MP}+\frac{MC}{MQ}$

Bài viết đã được chỉnh sửa nội dung bởi yeutoan89: 02-05-2018 - 21:26


#235
thanhdatnguyen2003

thanhdatnguyen2003

    Hạ sĩ

  • Thành viên
  • 68 Bài viết

Bài này làm kiểu gì mọi người: 

   

       Cho tam giác ABC có góc A không đổi và cạnh BC không đổi. Chứng minh chu vi tam giác ABC lớn nhất khi tam giác ABC cân


Bài viết đã được chỉnh sửa nội dung bởi thanhdatnguyen2003: 14-05-2018 - 15:22


#236
vkhoa

vkhoa

    Trung úy

  • Điều hành viên THPT
  • 933 Bài viết

Bài này làm kiểu gì mọi người: 

   

       Cho tam giác ABC có góc A không đổi và cạnh BC không đổi. Chứng minh chu vi tam giác ABC lớn nhất khi tam giác ABC cân

Góc $\widehat A$ không đổi nên $A$ chạy trên đ tròn cố định tâm $O$ đi qua 2 điểm $B, C$
gọi $D$ là điểm chính giữa cung $BC$ có chứa điểm $A$
không mất tính tổng quát, giả sử $A$ nằm trên cung nhỏ $BD$
$\Rightarrow AB <DB =DC$
$AC$ cắt $BD$ tại $E$
có $\triangle AEB\sim\triangle DEC$ (g, g)
$\Rightarrow\frac{AE}{DE} =\frac{AB}{DC} =\frac{BE}{CE} =\frac{AE +AB}{DE +DC}$
$\Leftrightarrow\frac{BE}{CE -BE} =\frac{AE +AB}{DE +DC -AE -AB}$
ta có $BE <AE +AB$
$\Rightarrow CE -BE <DE +DC -AE -AB$
$\Leftrightarrow CE +AE +AB <DE +DC +BE$
$\Leftrightarrow AC +AB <DB +DC$
$\Leftrightarrow AB +BC +CA <DB +BC +CD$
$\Leftrightarrow$ chu vi $\triangle ABC$ <chu vi $\triangle DBC$ (đpcm)

Hình gửi kèm

  • Cho tam giác ABC có góc A không đổi và cạnh BC không đổi. Chứng minh chu vi tam giác ABC lớn nhất khi tam giác ABC cân.png


#237
thanhdatnguyen2003

thanhdatnguyen2003

    Hạ sĩ

  • Thành viên
  • 68 Bài viết

 

Bài 110: cho tam giác ABC nhọn(AB<AC) nội tiếp đường tròn tâm (O). Gọi I là tâm đường tròn nội tiếp tam giác ABC. Tia AI cắt (O) tại J khác A. Đường thẳng JO cắt (O) tại K khác J và cắt BC tại E.

a.Chứng minh rằng J là tâm đường tròn ngoại tiếp tam giác IBC và JE.JK=JI2

b.Tiếp tuyến của (O) tại B và C cắt nhau tại S. Chứng minh rằng SJ.EK=SK.EJ
c.Đường thẳng SA cắt (O) tại D khác A ,đường thẳng DI cắt (O) tại M khác D
.Chứng minh rằng JM đi qua trung điểm của đoạn thẳng IE .
Bài 111: Cho tứ giác lồi ABCD . Gọi M,N lần lượt là trung điểm của AD và
BC ;AN cắt BM tại P , DN cắt CM tại Q .Tìm giá trị nhỏ nhất của biểu thức
$\ \frac{NA}{NP}+\frac{ND}{NQ}+\frac{MB}{MP}+\frac{MC}{MQ}$

 

bài này là đề thi chuyên bà rịa vũng tàu mà



#238
Grey Rabbit

Grey Rabbit

    Lính mới

  • Thành viên mới
  • 5 Bài viết

Cho em hỏi là tại sao lại nghĩ đến việc vẽ (HBC) ạ ? Bởi em thấy nó không được tự nhiên lắm

attachicon.gifHình.png

Vẽ đường tròn $(I)$ ngoại tiếp $\Delta AHB$ cắt $AB$ và $AC$ lần lượt tại $M$ và $N$. Vẽ đường tròn $(S)$ ngoại tiếp $\Delta AMN$. 

Gọi $H'$ là giao điểm của $SI$ và đường tròn $(I)$. 

Từ $\text{Bài Toán 48}$ ta có được bổ đề sau: Tứ giác $AOBS$ là hình bình hành 

Dễ dàng chứng minh được $H'G \parallel AD$ , mà $HG \parallel AD$ $\Rightarrow H \equiv H'$

$\Rightarrow HG$ là phân giác góc $\angle BHC$


RABBBBBBBBBBBBBBBBBBBBBBBBBBBBBBBBIT

FOOOOOOOOOOOOOOOOOOOOOOOOOOOOOD

 


#239
thuha1602

thuha1602

    Lính mới

  • Thành viên mới
  • 1 Bài viết

Giải giúp mình bài này với, Mình đang cần gấp. Xin cảm ơn:

 

Cho tam giác ABC, điểm E thuộc CA, điểm F thuộc AB, sao cho CE=BF. BE cắt CF tại P. M,N là tâm nội tiếp, BFP và PEC. Chứng minh MP=NQ, Q là giao phân giác góc A với MN



#240
maivanly0907

maivanly0907

    Lính mới

  • Thành viên mới
  • 2 Bài viết

BÀI 112: (THI THỬ KHTN VÒNG 1 NĂM 2019)

 

Cho tam giác ABC vuông cân tại A và nội tiếp trong đường trong (O).Dựng hình vuông BCEF sao cho E,F,A cùng phía với BC. Gọi giao điểm của OE,OF lần lượt với CA, AB là M,N. Gọi K,L là hình chiếu vuông góc của M,N xuống BC.

a) CMR: KMNL là hình vuông

b) Tia NM cắt (O) tại P. Chứng minh $PN.PM=MN^2$

c) Chứng minh rằng EP và FO cắt nhau tại một điểm thuộc (O)


Bài viết đã được chỉnh sửa nội dung bởi maivanly0907: 01-02-2024 - 21:34






Được gắn nhãn với một hoặc nhiều trong số những từ khóa sau: hình học

2 người đang xem chủ đề

0 thành viên, 2 khách, 0 thành viên ẩn danh